From time to time there is a public outcry against predatory pricing-where a company deliberately sells its products ...

REMI on September 29, 2020

A?

can someone explain why A is incorrect

Reply
Create a free account to read and take part in forum discussions.

Already have an account? log in

Shunhe on September 30, 2020

Hi,

Thanks for the question! So let’s take a look at the stimulus first. We’re told that occasionally, there’s public outcry against predatory pricing, which is when companies sell their products at super low prices to drive competitors out of business. So like if McDonald’s sold its burgers at a penny each to run Burger King out of business or something like that. But, the argument says, this argument clearly should be acceptable. Why? Because even after competitors go after business, the mere threat of renewed competition will prevent the company from jacking its prices up super high.

So now we’re asked for an assumption on which the argument depends. In other words, this is a strengthen with necessary premise question. We can use the negation test to test our answer choices. Let’s take a look at (A). Does the argument have to assume that any company that’s successful will inevitably induce competitors to enter the market? No, it doesn’t, and we can negate it to find out. Let’s say there are successful companies that don’t inevitably induce competitors to enter the market. Does that weaken the argument? No, it doesn’t. It doesn’t matter that the competitors will actually enter the market. What matters is that there’s a THREAT that they will enter the market. That’s what keeps the company from jacking up the prices. So negating (A) doesn’t weaken the argument, so it’s not a necessary assumption, and thus not the correct answer choice.

Hope this helps! Feel free to ask any other questions that you might have.